large pies cost £3.25 each
small pies cost £1.80 each

five children together buy 2 large pies and 1 small pie. they share the cost equally - how much does each child pay

Answers

Answer 1

Answer:

1.66 £

Step-by-step explanation:

(2 * 3.25 + 1.80) : 5 =

8.3 : 5

1.66 £


Related Questions

Which graph represents the solution of x2 + 9y2 ≤ 81 and y2 + 2 < x? On a coordinate plane, an ellipse has center (0, 0) and goes through (3, 0), (0, negative 9), (negative 3, 0), and (0, 9). A parabola opens to the right and goes through (6, 2), has vertex (2, 0), and goes through (6, negative 2). Everything inside of the ellipse and outside of the parabola is shaded. On a coordinate plane, an ellipse has center (0, 0) and goes through (3, 0), (0, negative 9), (negative 3, 0), and (0, 9). A parabola opens to the right and goes through (6, 2), has vertex (2, 0), and goes through (6, negative 2). Everything inside of the ellipse and inside of the parabola is shaded. On a coordinate plane, an ellipse has center (0, 0) and goes through (9, 0), (0, negative 3), (negative 9, 0) and (0, 3). A parabola opens to the right and goes through (6, 2), has vertex (2, 0), and goes through (6, negative 2). Everything inside of the ellipse and inside of the parabola is shaded. On a coordinate plane, an ellipse has center (0, 0) and goes through (9, 0), (0, negative 3), (negative 9, 0) and (0, 3). A parabola opens to the right and goes through (6, 2), has vertex (2, 0), and goes through (6, negative 2). Everything inside of the ellipse and outside of the parabola is shaded.

Answers

9514 1404 393

Answer:

  C

Step-by-step explanation:

The ellipse y-intercepts are ±3, the x-intercepts are ±9, eliminating choices A and B. The parabola is shaded inside, eliminating choice D.

The correct choice is the third one.

álgebra 1 solve -7 + 18(17h + 19)

Answers

Answer:  =360h+335

Step-by-step explanation:

Answer:

306h + 281

Step-by-step explanation:

-7 + 18(17h + 19)

-7 + 306h + 288

306h + 281

Find the measure of the missing angle using exterior angle sum theorm

Answers

35 degrees.

Assuming 145 is the exterior angle, the interior is 35. 180-145=35
Add 110 to 35 and get 145 degrees. Which leads the remaining angle 35 degrees

Answer:

35

Step-by-step explanation:

The exterior sum theorem states the exterior angle is equal to the sum of the opposite interior angles

145 = ?+110

145 - 100 = ?

35 = ?

One number is 6 times a second number. The sum of the two numbers is 56. Find the numbersOne number is 6 times a second number.

The sum of the two numbers is 56. Find the numbers

Answers

Answer:

I think its 56 = 56

Step-by-step explanation:

Answer:

8 and 48

Step-by-step explanation:

This can be written as a system of equations. X will be the smaller number and y will be the larger:

6x = y

x + y = 56

Then substitute y in the second equation for 6x(seen in first equation) and solve.

x + 6x = 56

7x = 56

x = 8

So one number is 8. To find the other plug in 8 for x in either of the original equations (both get the same answer for y).

6x = y

6 * 8 = y

48 = y

Then double check by seeing if they fit the requirements of the problem.

Hope this helps!

I forgot to label the triangle below! I just know that the cos A = 0.48. Based on this information, which angle should be marked A?

Answers

Answer:

angle 1

Step-by-step explanation:

Using the trigonometric mnemonic SOH CAH TOA, we know that cos or cosine is the ratio between the adjacent side and hypotenuse side.

This means that if cos A = 0.48, A is the measure of the angle which it's relative adjacent side divided by the hypotenuse of the triangle will be around 0.48.

Let's try angle 2, cos (angle 2) = adjacent / hypotenuse = 7.8 / 8.9 = 0.876404494382 ≈ 0.87 ≠ 0.48. Since the proportions are not equal, this angle cannot be the one marked as A.

Since angle 3 is a right angle, the adjacent could be either side so it cannot be correct. Thus angle 1 is correct.

Plis help me it’s for today

Answers

Answer:

Following are the solution to the given points:

Step-by-step explanation:

For question 1:

[tex]\to 3^{-4}= \frac{1}{3^4}=\frac{1}{81}=0.0123456789[/tex]

For question 2:

[tex]\to (-2)^{3}\cdot(-2)^{4}\cdot(-2)^{-1}=-8\cdot-16\cdot -\frac{1}{2}= 128\cdot -\frac{1}{2}=-64[/tex]

For question 3:

[tex]\to 7^{-4} \div 7^{-2}= \frac{1}{7^{4}} \div \frac{1}{7^{2}}=\frac{1}{7^{4}} \times \frac{7^{2}}{1}=\frac{1}{7^{2}} =\frac{1}{49} =0.0204081633[/tex]

For question 4:

[tex]\to [(-3)^{2}]^3= (-3)^{2\cdot 3}= (-3)^{6}=729[/tex]

For question 5:

[tex]\to [5 \cdot (-3)]^{2}= 25 \cdot 9=225[/tex]

For question 6:

[tex]\to [(10 \div 5)]^{3}= [(\frac{10}{5})]^{3}=[2]^{3}=8[/tex]

For question 7:

[tex]\to 10^6 \cdot 10^{-4} \cdot 10^2= 10^6 \cdot \frac{1}{10^{4}} \cdot 10^2= 10^2 \cdot 10^2=10^4=10,000[/tex]

For question 8:

[tex]\to (-4)^{-5}=\frac{1}{(-4)^{5}}=- \frac{1}{1,024}=-0.0009765625[/tex]

For question 9:

[tex]\to \frac{2^3}{2^4}= \frac{8}{16}=\frac{1}{2}=0.5[/tex]

For question 10:

[tex]\to (-6)^3 \cdot (-6)^5 \cdot (-6)^{-5}= (-6)^3 \cdot (-6)^5 \cdot \frac{1}{(-6)^{5}}= (-6)^3 =-216[/tex]

can someone answer plssss gives 100 pints i think bc I picked 100

Answers

[tex]1. \frac{20}{100} [/tex]

[tex]2. \frac{1}{5} [/tex]

Answer:  1

Explanation:

There are a few ways to do this. One way is to notice that the jump from 5 to 100 is "times 20" (go from right to left across the bottom denominators).

So we must do the same "times 20" type of jump when going across the numerators. If x is the numerator for the right hand side, then we go from x to 20. That must mean x = 1

Put another way, we could have these steps

20/100 = x/5

20*5 = 100*x ... cross multiplication

100 = 100x

100x = 100

x = 100/100 .... dividing both sides by 100

x = 1

We see that the fraction 20/100 reduces fully to 1/5

To go from 1/5 to 20/100, we multiply both parts by 20 (divide both parts by 20 to go in reverse).

helpppppppppppppppppp

Answers

Answer:

Dude

sheesh i only seeing 5 and 18
The answer is 5 it is the only number that can be multiplied by another number to get 10

Ariel has 20 bouncy balls. He later then sold them for a price of $2 per ball to his best friend, Mike Hawk. Later, Ariel has gotten back 1/69 of his original amount of bouncy balls and got 20 more. How many bouncy balls does Ariel have?

Answers

ariel has 40 balls left

Answer:

420

Step-by-step explanation:

Please help :(((((((((((((((((((((((((

Answers

jeez that looks really hard

Step-by-step explanation:

30 points if helped, question in the picture

Answers

Step-by-step explanation:

[tex] \frac{2}{2 + \sqrt{3} } \times \frac{2 - \sqrt{3} }{2 - \sqrt{3} } = \frac{2(2 - \sqrt{3} )}{1} = 4 - 2 \sqrt{3} [/tex]

a train leaves Westchester at 6:30. What time should it arrive at Middlewich

Answers

Answer:

a). 6:51

b). 6:30 am

c). 34 minutes

Step-by-step explanation:

a). Train leaves Westchester at 6:30.

   From the arrival - departure table in column (2),

   Arrival time of the train at middlewich = 6:51

b). Kate has to reach Southam before 9:00 am

   Therefore, time of the latest train that she can catch to get to work on time is 6:30 am

    By this train she can reach at 07:19 at Southam.

c). Duration of journey from Westchester to Eastwick = 06:34 - 06:00

                                                                                        = 00:34

                                                                                        ≈ 34 minutes

Instructions: Problem 2 ! Find the missing angle in the image below. Do not include spaces in your answers

Answers

Step-by-step explanation:

since angles in a triangle add up to 180

<vuw=180-(71+23)

=86°

since angles in a straight line add up to 180

<vuf=180-86

=94

Which best describes the vertex of the graph?


a (-3, -4)
b (-3, -4)
c (3, -4)
d (3, -4)

Answers

Answer: C

Step-by-step explanation:

C. Is the answer for sure

You invest $4.000 in a savings account. The account pays 3% annual interest. How much money will be in the savings account after 9 years?

A) $4,938.29
B) $5,219.20
C) $5,124.33
D) $6,003.45​

Answers

Answer:

4000*0.03*9=1080

4000+1080=5080

umm but like thats not one of the answers sorry

Hope This Helps!!!

Step-by-step explanation:

4000*3% = 120

120*9= 1080

1080+4000=5080

BRAINLIEST FOR ANSWER WITH EXPLANATION
JUST ONE QUESTION ATTACHED BELOW:

Answers

Answer:

m=16

Step-by-step explanation:

(6+6)/m=6/8

12/m=6/8

M=16

Answer:

16

Step-by-step explanation:

This triangle has been dialated; ILM is a dialated version of IJK. We know that dialations are by scale factors, and we also know the length of two corresponding sides.

IM is 6 m long, and IMK is 6+6 m long, so 12 m. So using simple ratios, we know the dialation factor from ILM to IJK is 2. The corresponding side for JK (m) is LM, which we know is 8 m. So side m is 16 because of the scale factor (2).

Maybe you thought we were solving for "m", as in a varriable all the sides are multiplied by, but I think that's just bad labeling. Hope this helps!

someone please help me ASAP!

Answers

Answer:

214°

Step-by-step explanation:

The measure of an arc that sees the center angle of the circle is equal to the very same angle that it sees

Since the measure of circle is 360° and arc AB is given as 146° the measure of arc ACB should be 360 - 146 = 214°

pls help me in these questions ​

Answers

Answer:

1=85

2=10

3=108

Step-by-step explanation:

Number 1: Calculate each angle...and you know a straight line is 180°. N the interior sum of a quadrilateral is always 360°.

Number2: Use corresponding, alternate and interior angles method. It will help

Number 3: It's just about solving the interior sum of the pentagon

Answer:

2. 10 degrees

3. 108 degrees

Step-by-step explanation:

sorry I reposted it because the previous answer was deleted and plz mark me as a brainliest.

Word problem One of the citizens has 97 silver coins. How many bronze coins would it take to equal this amount

Answers

Given: Given that a citizen have 97 silver coins.

To find : Here we need to find that how many bronze coins would it take to equal this amount.

Solution: We know, 1 silver coin=10 bronze coin

So, 97 silver coin=10×97 bronze coin

=970 bronze coin

Therefore, 970 bronze coins would it take to equal this amount.

Find the equation of the line through point (2,2) and parallel to y=x+4. Use a forward slash (i.e.”/“) for fractions (e.g. 1/2 for

Answers

Answer:

The equation of the line is, y = x

Step-by-step explanation:

The constraints of the required linear equation are;

The point through which the line passes = (2, 2)

The line to which the required line is parallel = y = x + 4

Two lines are parallel if they have the same slope, therefore, we have;

The slope of the line, y = x + 4 is m = 1

Therefore, the slope of the required line = 1

The equation of the required lime in point and slope form becomes;

y - 2 = 1 × (x - 2)

∴ y = x - 2 + 2 = x

The equation of the required line is therefore, y = x

if sam has 20 watermelons and he eats 14 how many does he have left?

Answers

Answer:

6 duh!

you can do this by yourself lol

please help!
factor f(x)=6x^2-x-2​

Answers

Hi there!

[tex]\large\boxed{(3x - 2)(2x + 1) }[/tex]

6x² - x - 2

To solve, we must factor into the following format:

(ax - b)(cx - d)

The following conditions must be met:

a · c = 6

da + bd = -1

b · d = -2

By guessing and checking, we get:

(3x - 2)(2x + 1)

Answer for Acellus:

(2 x + 1) (3 x - 2)

Step-by-step explanation:

i checked the equation on an online calculator, and it came up with it switched (compared to the other answer that is posted) like so...

I put the answer in and it said it was right for Acellus.

Hope this helps :))

please tel me answer of under root 3+4i
without calculatot with steps

Answers

Hello,

[tex]Let's\ say \\\\z=\sqrt{3+4*i} =a+b*i\\\\z^2=3+4*i=(a+b*i)^2=a^2-b^2+2i*a*b\\\\\\if \ a\neq 0\\\left\{\begin{array}{ccc}a^2+b^2&=&3\\2ab=4\\\end{array}\right.\\\\\\\left\{\begin{array}{ccc}b=\dfrac{2}{a}\\a^2-(\dfrac{2}{a})^2=2\\\end{array}\right.\\\\\\a^4-4=3*a^2\\a^4-3a^2-4=0\\\\\Delta=(-3)^2-4*1*(-4)=25=5^2\\\\a^2=4\ or \ a^2=-1 (impossible)\\\\So:\\(a=2\ and\ b=1)\ or\ (a=-2\ and\ b=-1)\\[/tex]

Roots are thus 2+i and -2-i

There is an other using a geometrical formula (formule de Moivre)

Hey Guys! Can You Please Help Me In My Math​

Answers

This would be a great tree diagram
Sorry...

pic is not clear.....

Surface area of a cuboid is 384
work out the volume

Answers

Answer:

is 512

Step-by-step explanation:

Can someone help me?

Answers

Answer:

C

Step-by-step explanation:

its asking for y, on the graph, the line is placed on point 4

If the quadratic formula is used to find the solution set of 3x + 4x-2 = 0, what are the solutions? ​

Answers

Answer is c
hope it helps I’m new at this

The number of clicks for a search text ad is 50 and the number of impressions is 5000. The CTR would be Group of answer choices 1% 2% 5% 10%

Answers

Given:

Clicks = 50

Impressions = 5000

To find:

The CTR percentage.

Solution:

We know that,

[tex]CTR=\dfrac{\text{Clicks}}{\text{Impressions}}\times 100[/tex]

Substituting the given values, we get

[tex]CTR=\dfrac{50}{5000}\times 100[/tex]

[tex]CTR=\dfrac{1}{100}\times 100[/tex]

[tex]CTR=1\%[/tex]

Therefore, the correct option is A.

Write the equation of the graph y=

Answers

9514 1404 393

Answer:

  y = 6^x -3

Step-by-step explanation:

The graph is that of an exponential function that has been translated downward. We notice the horizontal asymptote is -3, and a couple of points on the graph are (0, -2) and (1, 3).

The shifted parent function will look like ...

  y = a·b^x +c

where c is the horizontal asymptote. Using the two points we found, we have ...

  -2 = a·b^0 -3 . . . . . using (x, y) = (0, -2)

  1 = a . . . . . . . . . . add 3 and simplify

Then using (x, y) = (1, 3), we have ...

  3 = b^1 -3

  6 = b . . . . . . . . . add 3 and simplify

So, the equation is ...

  y = 6^x -3

Solve for x

Marking brainliest

Answers

Answer:

110°

Step-by-step explanation:

Other Questions
Choose the best graph that represents the linear equation:y + 3 = 0Graph AOn a coordinate plane, a line goes through (0, 3) and (1, 3).Graph BOn a coordinate plane, a line goes through (negative 3, 0) and (negative 3, 1).Graph COn a coordinate plane, a line goes through (0, negative 3) and (1, negative 3).Graph DOn a coordinate plane, a line goes through (0, 0) and (1, negative 3).a.Graph Ac.Graph Cb.Graph Bd.Graph DPLEASE HELP!!! Please select the best answer from the choices providedABCD Amy has 2$, Jack has 3 times as much as Amy. Catherine has twice as much as Jack. How much does Catherine have? M H To determine the number of deer in a game preserve, a conservationist catches 412 deer, tags them and lets them loose. Later, 316 deer are caught, 158 of them are tagged. How many deer are in the preserve? Please answer fast!!!!!!The sum of the reciprocals of two consecutive even integers is 9/40. This can be represented by the equation shown.1/x+1/x+2=9/40Use the rational equation to determine the integers. Show all work. what is the example of comparision with as...as Describe fully the single transformation that maps a onto shape b Calculate the vapor pressure (in torr) at 298 K in a solution prepared by dissolving 46.8 g of the non-volatile non-electrolye glucose in 117 g of methanol. The vapor pressure of methanol at 298 K is 122.7 torr. Enter your answer to 2 decimal places. What is the percent yield of the reaction below if 84.0 grams of Al2O3(s) is recovered from a reaction whose theoretical yield of Al2O3(s) is 104 grams?4 Al(s) + 3 O2(g) 2 Al2O3(s) A quality control expert at Glotech computers wants to test their new monitors. The production manager claims they have a mean life of 82 months with a standard deviation of 7 months. If the claim is true, what is the probability that the mean monitor life would be greater than 83.8 months in a sample of 71 monitors The obligation, placed on employees through delegation, to perform assigned tasks satisfactorily and be held accountable for the proper execution of work is the definition of ______. Solubility of which salts does not dependent on temperature.Any three. f) 1 5/2 +8x4 = alluda :( On Monday, 27 adults visited an amusement park. On Tuesday, 23 adults visited the amusement park. The enterance fee for the adults is Rs. 100. How much amount is collected from the adults in these two days? PLEASE TELL FULL SOLUTION. Calculate cos (theta) to two decimal places. PLS HELP ASAP PLZ HELPPP!!!!!!!!!! ASAP Based on the Punnett square, what is the probability that the offspring will have freckles?25 percent50 percent75 percent100 percent One of the first places sound effects were used was 3D movies radio plays dinosaur films on mixing boards Es un texto que ayuda a organizar cada una de las actividades que se llevarn a cabo dentro de un programa de radio qu es Which of the following did you include in your answer?Check all that apply.The Sun produces energy by the process ofnuclear fusion.Nuclear fusion occurs when lighter nuclei combineto produce a larger, heavier nucleus.Energy is released during nuclear fusion.Nuclear fusion requires very high temperatures andpressures.Nuclear fusion occurs in the core of the Sun whenhydrogen atoms combine to form helium atoms. Who was Bise Nagarchi? define.